GI System Full Set

अब Quizwiz के साथ अपने होमवर्क और परीक्षाओं को एस करें!

During a patient's scheduled home visit, an older adult patient has stated to the community health nurse that she has been experiencing hemorrhoids of increasing severity in recent months. The nurse should recommend which of the following? A) Regular application of an OTC antibiotic ointment B) Increased fluid and fiber intake C) Daily use of OTC glycerin suppositories D) Use of an NSAID to reduce inflammation

ANS: B Feedback: Hemorrhoid symptoms and discomfort can be relieved by good personal hygiene and by avoiding excessive straining during defecation. A high-residue diet that contains fruit and bran along with an increased fluid intake may be all the treatment that is necessary to promote the passage of soft, bulky stools to prevent straining. Antibiotics, regular use of suppositories, and NSAIDs are not recommended, as they do not address the etiology of the health problem.

A nurse is caring for an older adult who has been experiencing severe Clostridium difficile-related diarrhea. When reviewing the patient's most recent laboratory tests, the nurse should prioritize which of the following? A) White blood cell level B) Creatinine level C) Hemoglobin level D) Potassium level

ANS: D Feedback: In elderly patients, it is important to monitor the patient's serum electrolyte levels closely. Diarrhea is less likely to cause an alteration in white blood cell, creatinine, and hemoglobin levels.

A patient with GERD has undergone diagnostic testing and it has been determined that increasing the pace of gastric emptying may help alleviate symptoms. The nurse should anticipate that the patient may be prescribed what drug? A) Metoclopramide (Reglan) B) Omeprazole (Prilosec) C) Lansoprazole (Prevacid) D) Famotidine (Pepcid)

Ans: A Feedback: Metoclopramide (Reglan) is useful in promoting gastric motility. Omeprazole and lansoprozole are proton pump inhibitors that reduce gastric acid secretion. Famotidine (Pepcid) is an H2receptor antagonist, which has a similar effect.

The nursing instructor informs a student nurse that a client she is caring for has a chronic neurologic condition that decreases the client's peristalsis. What nursing diagnosis is the most likely risk for this client? A) Constipation B) Diarrhea C) Deficient fluid volume D) Excessive fluid volume

Ans: A Feedback: Peristalsis is defined as the contractions of the circular and longitudinal muscles of the intestine. Decreased peristalsis will result in constipation because the movement of the fecal mass will occur at a slower rate and more fluid will be absorbed in the colon.

A patient comes to the clinic complaining of pain in the epigastric region. What assessment question during the health interview would most help the nurse determine if the patient has a peptic ulcer? A) Does your pain resolve when you have something to eat? B) Do over-the-counter pain medications help your pain? C) Does your pain get worse if you get up and do some exercise? D) Do you find that your pain is worse when you need to have a bowel movement?

Ans: A Feedback: Pain relief after eating is associated with duodenal ulcers. The pain of peptic ulcers is generally unrelated to activity or bowel function and may or may not respond to analgesics.

Diagnostic imaging and physical assessment have revealed that a patient with peptic ulcer disease has suffered a perforated ulcer. The nurse recognizes that emergency interventions must be performed as soon as possible in order to prevent the development of what complication? A) Peritonitis B) Gastritis C) Gastroesophageal reflux D) Acute pancreatitis

Ans: A Feedback: Perforation is the erosion of the ulcer through the gastric serosa into the peritoneal cavity without warning. Chemical peritonitis develops within a few hours of perforation and is followed by bacterial peritonitis. Gastritis, reflux, and pancreatitis are not acute complications of a perforated ulcer.

A patient returns to the unit after a neck dissection. The surgeon placed a Jackson Pratt drain in the wound. When assessing the wound drainage over the first 24 postoperative hours the nurse would notify the physician immediately for what? A) Presence of small blood clots in the drainage B) 60 mL of milky or cloudy drainage C) Spots of drainage on the dressings surrounding the drain D) 120 mL of serosanguinous drainage

Ans: B Feedback: Between 80 and 120 mL of serosanguineous secretions may drain over the first 24 hours. Milky drainage is indicative of a chyle fistula, which requires prompt treatment.

A client is on bedrest, and an enema has been ordered. In what position should the nurse position the client? A) Fowler's B) Sims' C) Prone D) Sitting

Ans: B Feedback: A reclining position on the left side (Sims' position) is recommended. The head may be slightly elevated, but Fowler's position should be avoided because the solution will remain in the rectum and expulsion will occur rapidly, resulting in minimal cleansing.

A young woman has just consumed a serving of ice cream pie and develops severe cramping and diarrhea. The school nurse suspects the woman is ... A) Allergic to sugar B) Lactose intolerant C) Experiencing infectious diarrhea D) Deficient in fiber

Ans: B Feedback: Many people have difficulty digesting lactose (the sugar contained in milk products). The breakdown of lactose into its component sugars, glucose and galactose, requires a sufficient quantity of the enzyme lactase in the small intestine. If a person is lactose-deficient, alterations of bowel elimination, including formation of gas, abdominal cramping, and diarrhea, can occur after ingestion of milk products.

A nurse caring for a patient who has had bariatric surgery is developing a teaching plan in anticipation of the patient's discharge. Which of the following is essential to include? A) Drink a minimum of 12 ounces of fluid with each meal. B) Eat several small meals daily spaced at equal intervals. C) Choose foods that are high in simple carbohydrates. D) Sit upright when eating and for 30 minutes afterward.

Ans: B Feedback: Due to decreased stomach capacity, the patient must consume small meals at intervals to meet nutritional requirements while avoiding a feeling of fullness and complications such as dumping syndrome. The patient should not consume fluids with meals and low-Fowler's positioning is recommended during and after meals. Carbohydrates should be limited.

A nurse is providing health promotion education to a patient diagnosed with an esophageal reflux disorder. What practice should the nurse encourage the patient to implement? A) Keep the head of the bed lowered. B) Drinka cup of hot tea before bedtime. C) Avoid carbonated drinks. D) Eat a low-protein diet.

Ans: C Feedback: For a patient diagnosed with esophageal reflux disorder, the nurse should instruct the patient to keep the head of the bed elevated. Carbonated drinks, caffeine, and tobacco should be avoided. Protein limitation is not necessary.

A hospitalized toddler, previously bowel trained, has been having incontinent stools. What would the nurse tell the parents about this behavior? A) "When he does this, scold him and he will quit." B) "I don't understand why this child is losing control." C) "This is normal when a child this age is hospitalized." D) "I will have to call the doctor and report this behavior."

Ans: C Feedback: Discourage the use of punishment or shame for elimination accidents. Toddlers who are toilet trained often regress and experience soiling when hospitalized, and scolding or acting disgusted only reinforces the behavior.

A young woman comes to the emergency department with severe abdominal cramping and frequent bloody stools. Food poisoning is suspected. What diagnostic test would be used to confirm this diagnosis? A) Routine urinalysis B) Chest x-ray C) Stool sample D) Sputum sample

Ans: C Feedback: Outbreaks of food poisoning can result in severe gastrointestinal symptoms. Severe abdominal cramping followed by watery or bloody diarrhea may signal a microbial infection, which can be confirmed by a stool sample.

A patient is receiving education about his upcoming Billroth I procedure (gastroduodenostomy). This patient should be informed that he may experience which of the following adverse effects associated with this procedure? A) Persistent feelings of hunger and thirst B) Constipation or bowel incontinence C) Diarrhea and feelings of fullness D) Gastric reflux and belching

Ans: C Feedback: Following a Billroth I, the patient may have problems with feelings of fullness, dumping syndrome, and diarrhea. Hunger and thirst, constipation, and gastric reflux are not adverse effects associated with this procedure.

A nurse is preparing to provide care for a patient whose exacerbation of ulcerative colitis has required hospital admission. During an exacerbation of this health problem, the nurse would anticipate that the patient's stools will have what characteristics? A) Watery with blood and mucus B) Hard and black or tarry C) Dry and streaked with blood D) Loose with visible fatty streaks

ANS: A Feedback: The predominant symptoms of ulcerative colitis are diarrhea and abdominal pain. Stools may be bloody and contain mucus. Stools are not hard, dry, tarry, black or fatty in patients who have ulcerative colitis.

A patient has received treatment for oral cancer. The combination of medications and radiotherapy has resulted in leukopenia. Which of the following is an appropriate response to this change in health status? A) Ensure that none of the patient's visitors has an infection. B) Arrange for a diet that is high in protein and low in fat. C) Administer colony stimulating factors (CSFs) as ordered. D) Prepare to administer chemotherapeutics as ordered.

Ans: A Feedback: Leukopenia reduces defense mechanisms, increasing the risk of infections. Visitors who might transmit microorganisms are prohibited if the patients immunologic system is depressed. Changes in diet, CSFs, and the use of chemotherapy do not resolve leukopenia.

A nurse is documenting the appearance of feces from a client with a permanent ileostomy. Which of the following would she document? A) "Ileostomy bag half filled with liquid feces." B) "Ileostomy bag half filled with hard, formed feces." C) "Colostomy bag intact without feces." D) "Colostomy bag filled with flatus and feces."

Ans: A Feedback: The client with an ileostomy (temporary or permanent) has an opening into the small intestine. Because feces do not reach the large intestine, water is not absorbed, and the feces will be liquid.

A nurse assesses the abdomen of a client before and after administering a small-volume cleansing enema. What condition would be an expected finding? A) Increased bowel sounds B) Abdominal tenderness C) Areas of distention D) Muscular resistance

Ans: A Feedback: The goal of a cleansing enema is to increase peristalsis, which should increase bowel sounds.

A patient has received a diagnosis of gastric cancer and is awaiting a surgical date. During the preoperative period, the patient should adopt what dietary guidelines? A) Eat small, frequent meals with high calorie and vitamin content. B) Eat frequent meals with an equal balance of fat, carbohydrates, and protein. C) Eat frequent, low-fat meals with high protein content. D) Try to maintain the pre-diagnosis pattern of eating.

Ans: A Feedback: The nurse encourages the patient to eat small, frequent portions of nonirritating foods to decrease gastric irritation. Food supplements should be high in calories, as well as vitamins A and C and iron, to enhance tissue repair.

A patient has been diagnosed with an esophageal diverticulum after undergoing diagnostic imaging. When taking the health history, the nurse should expect the patient to describe what sign or symptom? A) Burning pain on swallowing B) Regurgitation of undigested food C) Symptoms mimicking a heart attack D) Chronic parotid abscesses

Ans: B Feedback: An esophageal diverticulum is an outpouching of mucosa and submucosa that protrudes through the esophageal musculature. Food becomes trapped in the pouch and is frequently regurgitated when the patient assumes a recumbent position. The patient may experience difficulty swallowing; however, burning pain is not a typical finding. Symptoms mimicking a heart attack are characteristic of GERD. Chronic parotid abscesses are not associated with a diagnosis of esophageal diverticulum.

What is occult blood? A) Bright red visible blood B) Dark black visible blood C) Blood that contains mucus D) Blood that cannot be seen

Ans: D Feedback: Occult blood in the stool is blood that is hidden in the specimen or cannot be seen on gross examination. It can be detected with simple screening tests, such as a Hematest.

A nurse is caring for a patient admitted with symptoms of an anorectal infection; cultures indicate that the patient has a viral infection. The nurse should anticipate the administration of what drug? A) Acyclovir (Zovirax) B) Doxycycline (Vibramycin) C) Penicillin (penicillin D) Metronidazole (Flagyl)

ANS: A Feedback: Acyclovir (Zovirax) is often given to patients with viral anorectal infections. Doxycycline (Vibramycin) and penicillin (penicillin G) are drugs of choice for bacterial infections. Metronidazole (Flagyl) is used for other infections with a bacterial etiology; it is ineffective against viruses.

A teenage patient with a pilonidal cyst has been brought for care by her mother. The nurse who is contributing to the patient's care knows that treatment will be chosen based on what risk? A) Risk for infection B) Risk for bowel incontinence C) Risk for constipation D) Risk for impaired tissue perfusion

ANS: A Feedback: Pilonidal cysts frequently develop into an abscess, necessitating surgical repair. These cysts do not contribute to bowel incontinence, constipation, or impaired tissue perfusion.

A nurse caring for a patient with colorectal cancer is preparing the patient for upcoming surgery. The nurse administers cephalexin (Keflex) to the patient and explains what rationale? A) To treat any undiagnosed infections B) To reduce intestinal bacteria levels C) To reduce bowel motility D) To reduce abdominal distention postoperatively

ANS: B Feedback: Antibiotics such a kanamycin (Kantrex), neomycin (Mycifradin), and cephalexin (Keflex) are administered orally the day before surgery to reduce intestinal bacterial. Preoperative antibiotics are not given to treat undiagnosed infections, reduce motility, or prevent abdominal distention.

A nurse is caring for a patient with constipation whose primary care provider has recommended senna (Senokot) for the management of this condition. The nurse should provide which of the following education points? A) "Limit your fluid intake temporarily so you don't get diarrhea." B) "Avoid taking the drug on a long-term basis." C) "Make sure to take a multivitamin with each dose." D) "Take this on an empty stomach to ensure maximum effect."

ANS: B Feedback: Laxatives should not be taken on an ongoing basis in order to reduce the risk of dependence. Fluid should be increased, not limited, and there is no need to take each dose with a multivitamin. Senna does not need to be taken on an empty stomach.

An adult client with a history of dyspepsia has been diagnosed with chronic gastritis. The nurse health education should include what guidelines? Select all that apply A) Prepare for the possibility of surgery B) Take calcium gluconate as prescribed C) Adopt a low-residue diet D) Avoid non-steroid anti-inflammatories E) Avoid drinking alcohol

And: D, E ; Avoid non-steroid anti-inflammatories, Avoid drinking alcohol

A nurse is caring for a patient who has undergone neck resection with a radial forearm free flap. The nurse's most recent assessment of the graft reveals that it has a bluish color and that mottling is visible. What is the nurse's most appropriate action? A) Document the findings as being consistent with a viable graft. B) Promptly report these indications of venous congestion. C) Closely monitor the patient and reassess in 30 minutes. D) Reposition the patient to promote peripheral circulation.

Ans: B Feedback: A graft that is blue with mottling may indicate venous congestion. This finding constitutes a risk for tissue ischemia and necrosis; prompt referral is necessary.

A patient has been admitted to the hospital after diagnostic imaging revealed the presence of a gastric outlet obstruction (GOO). What is the nurse's priority intervention? A) Administration of antiemetics B) Insertion of an NG tube for decompression C) Infusion of hypotonic IV solution D) Administration of proton pump inhibitors as ordered

Ans: B Feedback: In treating the patient with gastric outlet obstruction, the first consideration is to insert an NG tube to decompress the stomach. This is a priority over fluid or medication administration.

A nurse is providing care for a patient who has a diagnosis of irritable bowel syndrome (IBS). When planning this patient's care, the nurse should collaborate with the patient and prioritize what goal? A) Patient will accurately identify foods that trigger symptoms. B) Patient will demonstrate appropriate care of his ileostomy. C) Patient will demonstrate appropriate use of standard infection control precautions. D) Patient will adhere to recommended guidelines for mobility and activity.

ANS: A Feedback: A major focus of nursing care for the patient with IBS is to identify factors that exacerbate symptoms. Surgery is not used to treat this health problem and infection control is not a concern that is specific to this diagnosis. Establishing causation likely is more important to the patient than managing physical activity.

A 35-year-old male patient presents at the emergency department with symptoms of a small bowel obstruction. In collaboration with the primary care provider, what intervention should the nurse prioritize? A) Insertion of a nasogastric tube B) Insertion of a central venous catheter C) Administration of a mineral oil enema D) Administration of a glycerin suppository and an oral laxative

ANS: A Feedback: Decompression of the bowel through a nasogastric tube is necessary for all patients with small bowel obstruction. Peripheral IV access is normally sufficient. Enemas, suppositories, and laxatives are not indicated if an obstruction is present.

A nurse is presenting an educational event to a local community group. When speaking about colorectal cancer, what risk factor should the nurse cite? A) High levels of alcohol consumption B) History of bowel obstruction C) History of diverticulitis D) Longstanding psychosocial stress

ANS: A Feedback: Risk factors include high alcohol intake; cigarette smoking; and high-fact, high-protein, low-fiber diet. Diverticulitis, obstruction, and stress are not noted as risk factors for colorectal cancer.

The nurse is providing care for a patient whose inflammatory bowel disease has necessitated hospital treatment. Which of the following would most likely be included in the patient's medication regimen? A) Anticholinergic medications 30 minutes before a meal B) Antiemetics on a PRN basis C) Vitamin B12 injections to prevent pernicious anemia D) Beta adrenergic blockers to reduce bowel motility

ANS: A Feedback: The nurse administers anticholinergic medications 30 minutes before a meal as prescribed to decrease intestinal motility and administers analgesics as prescribed for pain. Antiemetics, vitamin B12 injections and beta blockers do not address the signs, symptoms, or etiology of inflammatory bowel disease.

A patient is admitted to the medical unit with a diagnosis of intestinal obstruction. When planning this patient's care, which of the following nursing diagnoses should the nurse prioritize? A) Ineffective Tissue Perfusion Related to Bowel Ischemia B) Imbalanced Nutrition: Less Than Body Requirements Related to Impaired Absorption C) Anxiety Related to Bowel Obstruction and Subsequent Hospitalization D) Impaired Skin Integrity Related to Bowel Obstruction

ANS: A Feedback: When the bowel is completely obstructed, the possibility of strangulation and tissue necrosis (i.e., tissue death) warrants surgical intervention. As such, this immediate physiologic need is a nursing priority. Nutritional support and management of anxiety are necessary, but bowel ischemia is a more immediate threat. Skin integrity is not threatened.

A nurse is caring for a patient who has been admitted to the hospital with diverticulitis. Which of the following would be appropriate nursing diagnoses for this patient? Select all that apply. A) Acute Pain Related to Increased Peristalsis and GI Inflammation B) Activity Intolerance Related to Generalized Weakness C) Bowel Incontinence Related to Increased Intestinal Peristalsis D) Deficient Fluid Volume Related to Anorexia, Nausea, and Diarrhea E) Impaired Urinary Elimination Related to GI Pressure on the Bladder

ANS: A, B, D Feedback: Patients with diverticulitis are likely to experience pain and decreased activity levels, and are at risk of fluid volume deficit. The patient is unlikely to experience fecal incontinence and urinary function is not directly influenced.

A nurse is providing care for a patient whose recent colostomy has contributed to a nursing diagnosis of Disturbed Body Image Related to Colostomy. What intervention best addresses this diagnosis? A) Encourage the patient to conduct online research into colostomies. B) Engage the patient in the care of the ostomy to the extent that the patient is willing. C) Emphasize the fact that the colostomy was needed to alleviate a much more serious health problem. D) Emphasize the fact that the colostomy is temporary measure and is not permanent.

ANS: B Feedback: For some patients, becoming involved in the care of the ostomy helps to normalize it and enhance familiarity. Emphasizing the benefits of the intervention is unlikely to improve the patient's body image, since the benefits are likely already known. Online research is not likely to enhance the patient's body image and some ostomies are permanent.

A patient has had an ileostomy created for the treatment of irritable bowel disease and the patient is now preparing for discharge. What should the patient be taught about changing this device in the home setting? A) Apply antibiotic ointment as ordered after cleaning the stoma. B) Apply a skin barrier to the peristomal skin prior to applying the pouch. C) Dispose of the clamp with each bag change. D) Cleanse the area surrounding the stoma with alcohol or chlorhexidine.

ANS: B Feedback: Guidelines for changing an ileostomy appliance are as follows. Skin should be washed with soap and water, and dried. A skin barrier should be applied to the peristomal skin prior to applying the pouch. Clamps are supplied one per box and should be reused with each bag change. Topical antibiotics are not utilized, but an antifungal spray or powder may be used.

A nursing instructor is discussing hemorrhoids with the nursing class. Which patients would the nursing instructor identify as most likely to develop hemorrhoids? A) A 45-year-old teacher who stands for 6 hours per day B) A pregnant woman at 28 weeks' gestation C) A 37-year-old construction worker who does heavy lifting D) A 60-year-old professional who is under stress

ANS: B Feedback: Hemorrhoids commonly affect 50% of patients after the age of 50. Pregnancy may initiate hemorrhoids or aggravate existing ones. This is due to increased constipation during pregnancy. The significance of pregnancy is greater than that of standing, lifting, or stress in the development of hemorrhoids.

A nurse caring for a patient with a newly created ileostomy assesses the patient and notes that the patient has had not ostomy output for the past 12 hours. The patient also complains of worsening nausea. What is the nurse's priority action? A) Facilitate a referral to the wound-ostomy-continence (WOC) nurse. B) Report signs and symptoms of obstruction to the physician. C) Encourage the patient to mobilize in order to enhance motility. D) Contact the physician and obtain a swab of the stoma for culture.

ANS: B Feedback: It is important to report nausea and abdominal distention, which may indicate intestinal obstruction. This requires prompt medical intervention. Referral to the WOC nurse is not an appropriate short-term response, since medical treatment is necessary. Physical mobility will not normally resolve an obstruction. There is no need to collect a culture from the stoma, because infection is unrelated to this problem.

A patient admitted with inflammatory bowel disease asks the nurse for help with menu selections. What menu selection is most likely the best choice for this patient? A) Spinach B) Tofu C) Multigrain bagel D) Blueberries

ANS: B Feedback: Nutritional management of inflammatory bowel disease requires ingestion of a diet that is bland, low-residue, high-protein, and high-vitamin. Tofu meets each of the criteria. Spinach, multigrain bagels, and blueberries are not low-residue.

A nurse at an outpatient surgery center is caring for a patient who had a hemorrhoidectomy. What discharge education topics should the nurse address with this patient? A) The appropriate use of antibiotics to prevent postoperative infection B) The correct procedure for taking a sitz bath C) The need to eat a low-residue, low-fat diet for the next 2 weeks D) The correct technique for keeping the perianal region clean without the use of water

ANS: B Feedback: Sitz baths are usually indicated after perianal surgery. A low-residue, low-fat diet is not necessary and water is used to keep the region clean. Postoperative antibiotics are not normally prescribed.

The nurse is caring for a patient who is undergoing diagnostic testing for suspected malabsorption. When taking this patient's health history and performing the physical assessment, the nurse should recognize what finding as most consistent with this diagnosis? A) Recurrent constipation coupled with weight loss B) Foul-smelling diarrhea that contains fat C) Fever accompanied by a rigid, tender abdomen D) Bloody bowel movements accompanied by fecal incontinence

ANS: B Feedback: The hallmarks of malabsorption syndrome from any cause are diarrhea or frequent, loose, bulky, foul-smelling stools that have increased fat content and are often grayish (steatorrhea). Constipation and bloody bowel movements are not suggestive of malabsorption syndromes. Fever and a tender, rigid abdomen are associated with peritonitis.

An older adult who resides in an assisted living facility has sought care from the nurse because of recurrent episodes of constipation. Which of the following actions should the nurse first perform? A) Encourage the patient to take stool softener daily. B) Assess the patient's food and fluid intake. C) Assess the patient's surgical history. D) Encourage the patient to take fiber supplements.

ANS: B Feedback: The nurse should follow the nursing process and perform an assessment prior to interventions. The patient's food and fluid intake is more likely to affect bowel function than surgery.

A 16-year-old presents at the emergency department complaining of right lower quadrant pain and is subsequently diagnosed with appendicitis. When planning this patient's nursing care, the nurse should prioritize what nursing diagnosis? A) Imbalanced Nutrition: Less Than Body Requirements Related to Decreased Oral Intake B) Risk for Infection Related to Possible Rupture of Appendix C) Constipation Related to Decreased Bowel Motility and Decreased Fluid Intake D) Chronic Pain Related to Appendicitis

ANS: B Feedback: The patient with a diagnosis of appendicitis has an acute risk of infection related to the possibility of rupture. This immediate physiologic risk is a priority over nutrition and constipation, though each of these concerns should be addressed by the nurse. The pain associated with appendicitis is acute, not chronic.

A patient admitted with acute diverticulitis has experienced a sudden increase in temperature and complains of a sudden onset of exquisite abdominal tenderness. The nurse's rapid assessment reveals that the patient's abdomen is uncharacteristically rigid on palpation. What is the nurse's best response? A) Administer a Fleet enema as ordered and remain with the patient. B) Contact the primary care provider promptly and report these signs of perforation. C) Position the patient supine and insert an NG tube. D) Page the primary care provider and report that the patient may be obstructed.

ANS: B Feedback: The patient's change in status is suggestive of perforation, which is a surgical emergency. Obstruction does not have this presentation involving fever and abdominal rigidity. An enema would be strongly contraindicated. An order is needed for NG insertion and repositioning is not a priority.

A patient with a diagnosis of colon cancer is 2 days postoperative following bowel resection and anastomosis. The nurse has planned the patient's care in the knowledge of potential complications. What assessment should the nurse prioritize? A) Close monitoring of temperature B) Frequent abdominal auscultation C) Assessment of hemoglobin, hematocrit, and red blood cell levels D) Palpation of peripheral pulses and leg girth

ANS: B Feedback: After bowel surgery, it is important to frequently assess the abdomen, including bowel sounds and abdominal girth, to detect bowel obstruction. The resumption of bowel motility is a priority over each of the other listed assessments, even though each should be performed by the nurse.

An adult patient has been diagnosed with diverticular disease after ongoing challenges with constipation. The patient will be treated on an outpatient basis. What components of treatment should the nurse anticipate? Select all that apply. A) Anticholinergic medications B) Increased fiber intake C) Enemas on alternating days D) Reduced fat intake E) Fluid reduction

ANS: B, D Feedback: Patients whose diverticular disease does not warrant hospital treatment often benefit from a high-fiber, low-fat diet. Neither enemas nor anticholinergics are indicated, and fluid intake is encouraged.

A nurse is assessing a patient's stoma on postoperative day 3. The nurse notes that the stoma has a shiny appearance and a bright red color. How should the nurse best respond to this assessment finding? A) Irrigate the ostomy to clear a possible obstruction. B) Contact the primary care provider to report this finding. C) Document that the stoma appears healthy and well perfused. D) Document a nursing diagnosis of Impaired Skin Integrity.

ANS: C Feedback: A healthy, viable stoma should be shiny and pink to bright red. This finding does not indicate that the stoma is blocked or that skin integrity is compromised.

A patient's screening colonoscopy revealed the presence of numerous polyps in the large bowel. What principle should guide the subsequent treatment of this patient's health problem? A) Adherence to a high-fiber diet will help the polyps resolve. B) The patient should be assured that these are a normal, age-related physiologic change. C) The patient's polyps constitute a risk factor for cancer. D) The presence of polyps is associated with an increased risk of bowel obstruction.

ANS: C Feedback: Although most polyps do not develop into invasive neoplasms, they must be identified and followed closely. They are very common, but are not classified as a normal, age-related physiologic change. Diet will not help them resolve and they do not typically lead to obstructions.

A patient's health history is suggestive of inflammatory bowel disease. Which of the following would suggest Crohn's disease, rather that ulcerative colitis, as the cause of the patient's signs and symptoms? A) A pattern of distinct exacerbations and remissions B) Severe diarrhea C) An absence of blood in stool D) Involvement of the rectal mucosa

ANS: C Feedback: Bloody stool is far more common in cases of UC than in Crohn's. Rectal involvement is nearly 100% in cases of UC (versus 20% in Crohn's) and patients with UC typically experience severe diarrhea. UC is also characterized by a pattern of remissions and exacerbations, while Crohn's often has a more prolonged and variable course.

A nurse is planning discharge teaching for a 21-year-old patient with a new diagnosis of ulcerative colitis. When planning family assessment, the nurse should recognize that which of the following factors will likely have the greatest impact on the patient's coping after discharge? A) The family's ability to take care of the patient's special diet needs B) The family's ability to monitor the patient's changing health status C) The family's ability to provide emotional support D) The family's ability to manage the patient's medication regimen

ANS: C Feedback: Emotional support from the family is key to the patient's coping after discharge. A 21-year-old would be expected to self-manage the prescribed medication regimen and the family would not be primarily responsible for monitoring the patient's health status. It is highly beneficial if the family is willing and able to accommodate the patient's dietary needs, but emotional support is paramount and cannot be solely provided by the patient alone.

A patient's colorectal cancer has necessitated a hemicolectomy with the creation of a colostomy. In the 4 days since the surgery, the patient has been unwilling to look at the ostomy or participate in any aspects of ostomy care. What is the nurse's most appropriate response to this observation? A) Ensure that the patient knows that he or she will be responsible for care after discharge. B) Reassure the patient that many people are fearful after the creation of an ostomy. C) Acknowledge the patient's reluctance and initiate discussion of the factors underlying it. D) Arrange for the patient to be seen by a social worker or spiritual advisor.

ANS: C Feedback: If the patient is reluctant to participate in ostomy care, the nurse should attempt to dialogue about this with the patient and explore the factors that underlie it. It is presumptive to assume that the patient's behavior is motivated by fear. Assessment must precede referrals and emphasizing the patient's responsibilities may or may not motivate the patient.

A nurse is working with a patient who is learning to care for a continent ileostomy (Kock pouch). Following the initial period of healing, the nurse is teaching the patient how to independently empty the ileostomy. The nurse should teach the patient to do which of the following actions? A) Aim to eventually empty the pouch every 90 minutes. B) Avoid emptying the pouch until it is visibly full. C) Insert the catheter approximately 5 cm into the pouch. D) Aspirate the contents of the pouch using a 60 mL piston syringe.

ANS: C Feedback: To empty a Kock pouch, the catheter is gently inserted approximately 5 cm to the point of the valve or nipple. The length of time between drainage periods is gradually increased until the reservoir needs to be drained only every 4 to 6 hours and irrigated once each day. It is not appropriate to wait until the pouch is full, and this would not be visible. The contents of the pouch are not aspirated.

A nurse is working with a patient who has chronic constipation. What should be included in patient teaching to promote normal bowel function? A) Use glycerin suppositories on a regular basis. B) Limit physical activity in order to promote bowel peristalsis. C) Consume high-residue, high-fiber foods. D) Resist the urge to defecate until the urge becomes intense.

ANS: C Feedback: Goals for the patient include restoring or maintaining a regular pattern of elimination by responding to the urge to defecate, ensuring adequate intake of fluids and high-fiber foods, learning about methods to avoid constipation, relieving anxiety about bowel elimination patterns, and avoiding complications. Ongoing use of pharmacologic aids should not be promoted, due to the risk of dependence. Increased mobility helps to maintain a regular pattern of elimination. The urge to defecate should be heeded.

An older adult has a diagnosis of Alzheimer's disease and has recently been experiencing fecal incontinence. However, the nurse has observed no recent change in the character of the patient's stools. What is the nurse's most appropriate intervention? A) Keep a food diary to determine the foods that exacerbate the patient's symptoms. B) Provide the patient with a bland, low-residue diet. C) Toilet the patient on a frequent, scheduled basis. D) Liaise with the primary care provider to obtain an order for loperamide.

ANS: C Feedback: Because the patient's fecal incontinence is most likely attributable to cognitive decline, frequent toileting is an appropriate intervention. Loperamide is unnecessary in the absence of diarrhea. Specific foods are not likely to be a cause of, or solution to, this patient's health problem.

A nurse is talking with a patient who is scheduled to have a hemicolectomy with the creation of a colostomy. The patient admits to being anxious, and has many questions concerning the surgery, the care of a stoma, and necessary lifestyle changes. Which of the following nursing actions is most appropriate? A) Reassure the patient that the procedure is relatively low risk and that patients are usually successful in adjusting to an ostomy. B) Provide the patient with educational materials that match the patient's learning style. C) Encourage the patient to write down these concerns and questions to bring forward to the surgeon. D) Maintain an open dialogue with the patient and facilitate a referral to the wound-ostomy-continence (WOC) nurse.

ANS: D Feedback: A wound-ostomy-continence (WOC) nurse is a registered nurse who has received advanced education in an accredited program to care for patients with stomas. The enterostomal nurse therapist can assist with the selection of an appropriate stoma site, teach about stoma care, and provide emotional support. The surgeon is less likely to address the patient's psychosocial and learning needs. Reassurance does not address the patient's questions, and education may or may not alleviate anxiety.

A patient has been experiencing disconcerting GI symptoms that have been worsening in severity. Following medical assessment, the patient has been diagnosed with lactose intolerance. The nurse should recognize an increased need for what form of health promotion? A) Annual screening colonoscopies B) Adherence to recommended immunization schedules C) Regular blood pressure monitoring D) Frequent screening for osteoporosis

ANS: D Feedback: Persons with lactose intolerance often experience hypocalcemia and a consequent risk of osteoporosis related to malabsorption of calcium. Lactose intolerance does not create an increased need for screening for colorectal cancer, immunizations, or blood pressure monitoring.

A patient has been experiencing occasional episodes of constipation and has been unable to achieve consistent relief by increasing physical activity and improving his diet. What pharmacologic intervention should the nurse recommend to the patient for ongoing use? A) Mineral oil enemas B) Bisacodyl (Dulcolax) C) Senna (Senokot) D) Psyllium hydrophilic mucilloid (Metamucil)

ANS: D Feedback: Psyllium hydrophilic mucilloid (Metamucil) is a bulk-forming laxative that is safe for ongoing use. None of the other listed laxatives should be used on an ongoing basis because of the risk of dependence.

A nurse is teaching a group of adults about screening and prevention of colorectal cancer. The nurse should describe which of the following as the most common sign of possible colon cancer? A) Development of new hemorrhoids B) Abdominal bloating and flank pain C) Unexplained weight gain D) Change in bowel habits

ANS: D Feedback: The most common presenting symptom associated with colorectal cancer is a change in bowel habits. The passage of blood is the second most common symptom. Symptoms may also include unexplained anemia, anorexia, weight loss, and fatigue. Hemorrhoids and bloating are atypical.

Which of the following is the most plausible nursing diagnosis for a patient whose treatment for colon cancer has necessitated a colonostomy? A) Risk for Unstable Blood Glucose Due to Changes in Digestion and Absorption B) Unilateral Neglect Related to Decreased Physical Mobility C) Risk for Excess Fluid Volume Related to Dietary Changes and Changes In Absorption D) Ineffective Sexuality Patterns Related to Changes in Self-Concept

ANS: D Feedback: The presence of an ostomy frequently has an effect on sexuality; this should be addressed thoughtfully in nursing care. None of the other listed diagnoses reflects the physiologic changes that result from colorectal surgery.

A patient has been diagnosed with a small bowel obstruction and has been admitted to the medical unit. The nurse's care should prioritize which of the following outcomes? A) Preventing infection B) Maintaining skin and tissue integrity C) Preventing nausea and vomiting D) Maintaining fluid and electrolyte balance

ANS: D Feedback: All of the listed focuses of care are important for the patient with a small bowel obstruction. However, the patient's risk of fluid and electrolyte imbalances is an immediate threat to safety, and is a priority in nursing assessment and interventions.

A patient's large bowel obstruction has failed to resolve spontaneously and the patient's worsening condition has warranted admission to the medical unit. Which of the following aspects of nursing care is most appropriate for this patient? A) Administering bowel stimulants as ordered B) Administering bulk-forming laxatives as ordered C) Performing deep palpation as ordered to promote peristalsis D) Preparing the patient for surgical bowel resection

ANS: D Feedback: The usual treatment for a large bowel obstruction is surgical resection to remove the obstructing lesion. Administration of laxatives or bowel stimulants are contraindicated if the bowel is obstructed. Palpation would be painful and has no therapeutic benefit.

A patient has been diagnosed with achalasia based on his history and diagnostic imaging results. The nurse should identify what risk diagnosis when planning the patient's care? A) Risk for Aspiration Related to Inhalation of Gastric Contents B) Risk for Imbalanced Nutrition: Less than Body Requirements Related to Impaired Absorption C) Risk for Decreased Cardiac Output Related to Vasovagal Response D) Risk for Impaired Verbal Communication Related to Oral Trauma

Ans: A Feedback: Achalasia can result in the aspiration of gastric contents. It is not normally an acute risk to the patients nutritional status and does not affect cardiac output or communication.

The school nurse is planning a health fair for a group of fifth graders and dental health is one topic that the nurse plans to address. What would be most likely to increase the risk of tooth decay? A) Organic fruit juice B) Roasted nuts C) Red meat that is high in fat D) Cheddar cheese

Ans: A Feedback: Dental caries may be prevented by decreasing the amount of sugar and starch in the diet. Patients who snack should be encouraged to choose less cariogenic alternatives, such as fruits, vegetables, nuts, cheeses, or plain yogurt. Fruit juice is high in sugar, regardless of whether it is organic.

A nurse is caring for a patient who is postoperative from a neck dissection. What would be the most appropriate nursing action to enhance the patient's appetite? A) Encourage the family to bring in the patient's favored foods. B) Limit visitors at mealtimes so that the patient is not distracted. C) Avoid offering food unless the patient initiates. D) Provide thorough oral care immediately after the patient eats.

Ans: A Feedback: Family involvement and home-cooked favorite foods may help the patient to eat. Having visitors at mealtimes may make eating more pleasant and increase the patients appetite. The nurse should not place the complete onus for initiating meals on the patient. Oral care after meals is necessary, but does not influence appetite.

A patient's neck dissection surgery resulted in damage to the patient's superior laryngeal nerve. What area of assessment should the nurse consequently prioritize? A) The patient's swallowing ability B) The patient's ability to speak C) The patient's management of secretions D) The patient's airway patency

Ans: A Feedback: If the superior laryngeal nerve is damaged, the patient may have difficulty swallowing liquids and food because of the partial lack of sensation of the glottis. Damage to this particular nerve does not inhibit speech and only affects management of secretions and airway patency indirectly.

A patient with gastroesophageal reflux disease (GERD) has a diagnosis of Barrett's esophagus with minor cell changes. Which of the following principles should be integrated into the patient's subsequent care? A) The patient will require an upper endoscopy every 6 months to detect malignant changes. B) Liver enzymes must be checked regularly, as H2 receptor antagonists may cause hepatic damage. C) Small amounts of blood are likely to be present in the stools and are not cause for concern. D) Antacids may be discontinued when symptoms of heartburn subside.

Ans: A Feedback: In the patient with Barretts esophagus, the cells lining the lower esophagus have undergone change and are no longer squamous cells. The altered cells are considered precancerous and are a precursor to esophageal cancer. In order to facilitate early detection of malignant cells, an upper endoscopy is recommended every 6 months. H2 receptor antagonists are commonly prescribed for patients withGERD; however, monitoring of liver enzymes is not routine. Stools that contain evidence of frank bleeding or that are tarry are not expected and should be reported immediately. When antacids are prescribed for patients with GERD, they should be taken as ordered whether or not the patient is symptomatic.

An elderly patient comes into the emergency department complaining of an earache. The patient and has an oral temperature of 100.2ºF and otoscopic assessment of the ear reveals a pearly gray tympanic membrane with no evidence of discharge or inflammation. Which action should the triage nurse take next? A) Palpate the patient's parotid glands to detect swelling and tenderness. B) Assess the temporomandibular joint for evidence of a malocclusion. C) Test the integrity of cranial nerve XII by asking the patient to protrude the tongue. D) Inspect the patient's gums for bleeding and hyperpigmentation.

Ans: A Feedback: Older adults and debilitated patients of any age who are dehydrated or taking medications that reduce saliva production are at risk for parotitis. Symptoms include fever and tenderness, as well as swelling of the parotid glands. Pain radiates to the ear. Pain associated with malocclusion of the temporomandibular joint may also radiate to the ears; however, a temperature elevation would not be associated with malocclusion. The 12th cranial nerve is not associated with the auditory system. Bleeding and hyperpigmented gums may be caused by pyorrhea or gingivitis. These conditions do not cause earache; fever would not be present unless the teeth were abscessed.

A nurse who provides care in an ambulatory clinic integrates basic cancer screening into admission assessments. What patient most likely faces the highest immediate risk of oral cancer? A) A 65-year-old man with alcoholism who smokes B) A 45-year-old woman who has type 1 diabetes and who wears dentures C) A 32-year-old man who is obese and uses smokeless tobacco D) A 57-year-old man with GERD and dental caries

Ans: A Feedback: Oral cancers are often associated with the use of alcohol and tobacco, which when used together have a synergistic carcinogenic effect. Most cases of oral cancers occur in people over the age of 60 and a disproportionate number of cases occur in men. Diabetes, dentures, dental caries, and GERD are not risk factors for oral cancer.

A medical nurse who is caring for a patient being discharged home after a radical neck dissection has collaborated with the home health nurse to develop a plan of care for this patient. What is a priority psychosocial outcome for a patient who has had a radical neck dissection? A) Indicates acceptance of altered appearance and demonstrates positive self-image B) Freely expresses needs and concerns related to postoperative pain management C) Compensates effectively for alteration in ability to communicate related to dysarthria D) Demonstrates effective stress management techniques to promote muscle relaxation

Ans: A Feedback: Since radical neck dissection involves removal of the sternocleidomastoid muscle, spinal accessory muscles, and cervical lymph nodes on one side of the neck, the patients appearance is visibly altered. The face generally appears asymmetric, with a visible neck depression; shoulder drop also occurs frequently. These changes have the potential to negatively affect self-concept and body image.Facilitating adaptation to these changes is a crucial component of nursing intervention. Patients who have had head and neck surgery generally report less pain as compared with other postoperative patients; however, the nurse must assess each individual patients level of pain and response to analgesics. Patients may experience transient hoarseness following a radical neck dissection; however, their ability to communicate is not permanently altered. Stress management is beneficial but would not be considered the priority in this clinical situation.

The nurse notes that a patient who has undergone skin, tissue, and muscle grafting following a modified radical neck dissection requires suctioning. What is the most important consideration for the nurse when suctioning this patient? A) Avoid applying suction on or near the suture line. B) Position patient on the non operative side with the head of the bed down. C) Assess the patient's ability to perform self-suctioning. D) Evaluate the patient's ability to swallow saliva and clear fluids.

Ans: A Feedback: The nurse should avoid positioning the suction catheter on or near the graft suture lines. Application of suction in these areas could damage the graft. Self-sectioning may be unsafe because the patient may damage the suture line. Following a modified radical neck dissection with graft, the patient is usually positioned with the head of the bed elevated to promote drainage and reduce edema. Assessing viability of the graft is important but is not part of the suctioning procedure and may delay initiating suctioning. Maintenance of a patent airway is a nursing priority. Similarly, the patients ability to swallow is an important assessment for the nurse to make; however, it is not directly linked to the patients need for suctioning.

A nurse is caring for a patient who has had surgery for oral cancer. When addressing the patient's long-term needs, the nurse should prioritize interventions and referrals with what goal? A) Enhancement of verbal communication B) Enhancement of immune function C) Maintenance of adequate social support D) Maintenance of fluid balance

Ans: A Feedback: Verbal communication may be impaired by radical surgery for oral cancer. Addressing this impairment often requires a long-term commitment. Immune function, social support, and fluid balance are all necessary, but communication is a priority issue for patients recovering from this type of surgery.

A nurse is assessing the stools of a breastfed baby. What is the appearance of normal stools for this baby? A) Yellow, loose, odorless B) Brown, paste-like, some odor C) Brown, formed, strong odor D) Black, semiformed, no odor

Ans: A Feedback: Breast-fed babies have more frequent stools, and the stools are yellow to golden and loose, usually with little odor. Breast-fed babies can normally have 2 to 10 stools per day.

While caring for an infant who is breast-fed, the nurse assesses the characteristics of the stools. What stool characteristics are expected in breast-fed infants? A) Golden yellow and loose B) Dark brown and firm C) Yellow-brown and pasty D) Green and mucusy

Ans: A Feedback: Breast-fed infants have more frequent stools, and the stools are yellow to golden, loose, and usually have little odor. With formula or cow's milk feedings, infants' stools vary from yellow to brown and are pasty in consistency.

Which is an expected outcome for a client undergoing a bowel training program? A) Have a soft, formed stool at regular intervals without a laxative. B) Continue to use laxatives, but use one less irritating to the rectum. C) Use oil-retention enemas on a regular basis for elimination. D) Have a formed stool at least twice a day for two weeks.

Ans: A Feedback: Clients who have chronic constipation and impaction, and those who are incontinent of stool, may benefit from a bowel training program. The purpose of this program is to manipulate factors within the client's control (such as exercise or fluid intake) to produce the elimination of a soft, formed stool at regular intervals without a laxative.

A nurse is scheduling diagnostic studies for client. Which test would be performed first? A) Fecal occult blood test B) Barium study C) Endoscopic exam D) Upper gastrointestinal series

Ans: A Feedback: Nurses are commonly involved in scheduling diagnostic studies when a client is to undergo multiple studies. They should follow a logical sequence when more than one test is required for accurate diagnosis; that is, fecal occult blood tests to detect gastrointestinal bleeding; barium studies to visualize gastrointestinal structures and reveal any inflammation, ulcers, tumors, strictures, or other lesions; and endoscopic examinations to visualize an abnormality, locate a source of bleeding, and if necessary, provide biopsy tissue samples.

A client tells the nurse that he takes laxatives every day but is still constipated. The nurse's response is based on: A) Habitual laxative use is the most common cause of chronic constipation. B) If laxatives are not effective, the client should begin to use enemas. C) A laxative that works by a different method should be used. D) Chronic constipation is nothing to be concerned about.

Ans: A Feedback: Occasional use of laxatives is not harmful for most people, but they should not become dependent on them. Although many people do take laxatives because they believe they are constipated, most are unaware that habitual use of laxatives is the most common cause of chronic constipation.

A nurse is assessing a client with constipation and severe rectal pain. Which of the following actions should the nurse perform to determine the presence of fecal impaction? A) Inserted a lubricated, gloved finger into the rectum. B) Obtain a sharp intestinal x-ray. C) Insert a lubricated rectal tube into the rectum. D) Administer an oil retention enema into the rectum.

Ans: A Feedback: The nurse should insert a lubricated, gloved finger into the rectum to determine the presence of fecal impaction. Fecal impaction occurs when a large, hardened mass of stool interferes with defecation. Obtaining a sharp intestinal x-ray is not a good idea because the barium retained in the intestine causes fecal impaction. Insertion of a rectal tube and administration of an oil retention enema are measures used to remove hardened stool, not assess it.

The nurse is assessing a client with abdominal complaints. The nurse performs deep palpation of the abdomen for which reason? A) Detect abdominal masses B) Determine abdominal firmness C) Assess softness of abdominal muscles D) Assess degree of abdominal distention

Ans: A Feedback: The purpose of the deep palpation is to detect abdominal masses. Light palpation of the abdomen helps to determine the firmness or softness of the abdominal muscles and the degree of abdominal distention.

A nurse is following a physician's order to irrigate the NG tube of a client. Which of the following is a recommended guideline in this procedure? A) Assist client to 30- to 45-degree position, unless this is contraindicated. B) Draw up 60 mL of saline solution (or amount indicated in the order or policy) into syringe. C) If Salem sump or double-lumen tube is used, make sure that syringe tip is placed in the blue air vent. D) If unable to irrigate the tube, reposition client and attempt irrigation again; inject 20 to 30 mL of air and aspirate again.

Ans: A Feedback: To irrigate an NG tube, assist the client to 30- to 45-degree position, unless this is contraindicated. Pour the irrigating solution into the container and draw up 30 mL of saline solution (or amount indicated in the order or policy) into syringe. If Salem sump or double-lumen tube is used, make sure that syringe tip is placed in the drainage port and not in the blue air vent. If unable to irrigate the tube, reposition the client and attempt irrigation again. Inject 10 to 20 mL of air and aspirate again.

What are two essential techniques when collecting a stool specimen? A) Hand hygiene and wearing gloves B) Following policies and selecting containers C) Wearing goggles and an isolation gown D) Using a no-touch method and toilet paper

Ans: A Feedback: Use of medical aseptic techniques when collecting a stool specimen is imperative. Hand hygiene, before and after wearing rubber gloves, is essential.

During defecation, the client experiences decreased cardiac output related to the Valsalva maneuver. After the Valsalva maneuver, the nurse assesses the client's vital signs and expects to observe which of the following? A) An increase in the client's blood pressure B) A decrease in the client's blood pressure C) An increase in the client's respiratory rate D) A decrease in the client's respiratory rate

Ans: A Feedback: When an individual bears down to defecate, the increased pressures in the abdominal and thoracic cavities result in a decreased blood flow and a temporary decrease in cardiac output. Once the bearing down ceases, the pressure is lessened, and a larger than normal amount of blood returns to the heart; this act elevates the client's blood pressure.

A patient with a diagnosis of peptic ulcer disease has just been prescribed omeprazole (Prilosec). How should the nurse best describe this medication's therapeutic action? A) This medication will reduce the amount of acid secreted in your stomach. B) This medication will make the lining of your stomach more resistant to damage. C) This medication will specifically address the pain that accompanies peptic ulcer disease. D) This medication will help your stomach lining to repair itself.

Ans: A Feedback: Proton pump inhibitors like Prilosec inhibit the synthesis of stomach acid. PPIs do not increase the durability of the stomach lining, relieve pain, or stimulate tissue repair.

A patient was treated in the emergency department and critical care unit after ingesting bleach. What possible complication of the resulting gastritis should the nurse recognize? A) Esophageal or pyloric obstruction related to scarring B) Uncontrolled proliferation of H. pylori C) Gastric hyperacidity related to excessive gastrin secretion D) Chronic referred pain in the lower abdomen

Ans: A Feedback: A severe form of acute gastritis is caused by the ingestion of strong acid or alkali, which may cause the mucosa to become gangrenous or to perforate. Scarring can occur, resulting in pyloric stenosis (narrowing or tightening) or obstruction. Chronic referred pain to the lower abdomen is a symptom of peptic ulcer disease, but would not be an expected finding for a patient who has ingested a corrosive substance. Bacterial proliferation and hyperacidity would not occur.

A patient is one month postoperative following restrictive bariatric surgery. The patient tells the clinic nurse that he has been having "trouble swallowing" for the past few days. What recommendation should the nurse make? A) Eating more slowly and chewing food more thoroughly B) Taking an OTC antacid or drinking a glass of milk prior to each meal C) Chewing gum to cause relaxation of the lower esophageal sphincter D) Drinking at least 12 ounces of liquid with each meal

Ans: A Feedback: Dysphagia may be prevented by educating patients to eat slowly, to chew food thoroughly, and to avoid eating tough foods such as steak or dry chicken or doughy bread. After bariatric procedures, patients should normally not drink beverages with meals. Medications or chewing gum will not alleviate this problem.

A patient presents to the walk-in clinic complaining of vomiting and burning in her mid-epigastria. The nurse knows that in the process of confirming peptic ulcer disease, the physician is likely to order a diagnostic test to detect the presence of what? A) Infection with Helicobacter pylori B) Excessive stomach acid secretion C) An incompetent pyloric sphincter D) A metabolic acidbase imbalance

Ans: A Feedback: H. pylori infection may be determined by endoscopy and histologic examination of a tissue specimen obtained by biopsy, or a rapid urease test of the biopsy specimen. Excessive stomach acid secretion leads to gastritis; however, peptic ulcers are caused by colonization of the stomach by H. pylori. Sphincter dysfunction and acidbase imbalances do not cause peptic ulcer disease.

A patient has come to the clinic complaining of pain just above her umbilicus. When assessing the patient, the nurse notes Sister Mary Joseph's nodules. The nurse should refer the patient to the primary care provider to be assessed for what health problem? A) A GI malignancy B) Dumping syndrome C) Peptic ulcer disease D) Esophageal/gastric obstruction

Ans: A Feedback: Palpable nodules around the umbilicus, called Sister Mary Joseph's nodules, are a sign of a GI malignancy, usually a gastric cancer. This would not be a sign of dumping syndrome, peptic ulcer disease, or esophageal/gastric obstruction.

A patient with gastritis required hospital treatment for an exacerbation of symptoms and receives a subsequent diagnosis of pernicious anemia due to malabsorption. When planning the patient's continuing care in the home setting, what assessment question is most relevant? A) "Does anyone in your family have experience at giving injections?" B) "Are you going to be anywhere with strong sunlight in the next few months?" C) "Are you aware of your blood type?" D) "Do any of your family members have training in first aid?"

Ans: A Feedback: Patients with malabsorption of vitamin B12 need information about lifelong vitamin B12 injections; the nurse may instruct a family member or caregiver how to administer the injections or make arrangements for the patient to receive the injections from a health care provider. Questions addressing sun exposure, blood type and first aid are not directly relevant.

A nurse in the postanesthesia care unit admits a patient following resection of a gastric tumor. Following immediate recovery, the patient should be placed in which position to facilitate patient comfort and gastric emptying? A) Fowlers B) Supine C) Left lateral D) Left Sims

Ans: A Feedback: Positioning the patient in a Fowler's position postoperatively promotes comfort and facilitates emptying of the stomach following gastric surgery. Any position that involves lying down delays stomach emptying and is not recommended for this type of patient. Supine positioning and the left lateral (left Sim's) position do not achieve this goal.

A nurse is performing the admission assessment of a patient whose high body mass index (BMI) corresponds to class III obesity. In order to ensure empathic and patient-centered care, the nurse should do which of the following? A) Examine ones own attitudes towards obesity in general and the patient in particular. B) Dialogue with the patient about the lifestyle and psychosocial factors that resulted in obesity. C) Describe ones own struggles with weight gain and weight loss to the patient. D) Elicit the patients short-term and long-term goals for weight loss.

Ans: A Feedback: Studies suggest that health care providers, including nurses, harbor negative attitudes towards obese patients. Nurses have a responsibility to examine these attitudes and change them accordingly. This is foundational to all other areas of assessing this patient.

A patient who experienced an upper GI bleed due to gastritis has had the bleeding controlled and the patient's condition is now stable. For the next several hours, the nurse caring for this patient should assess for what signs and symptoms of recurrence? A) Tachycardia, hypotension, and tachypnea B) Tarry, foul-smelling stools C) Diaphoresis and sudden onset of abdominal pain D) Sudden thirst, unrelieved by oral fluid administration

Ans: A Feedback: Tachycardia, hypotension, and tachypnea are signs of recurrent bleeding. Patients who have had one GI bleed are at risk for recurrence. Tarry stools are expected short-term findings after a hemorrhage. Hemorrhage is not normally associated with sudden thirst or diaphoresis.

A nurse is presenting a class at a bariatric clinic about the different types of surgical procedures offered by the clinic. When describing the implications of different types of surgeries, the nurse should address which of the following topics? Select all that apply. A) Specific lifestyle changes associated with each procedure B) Implications of each procedure for eating habits C) Effects of different surgeries on bowel function D) Effects of various bariatric surgeries on fertility E) Effects of different surgeries on safety of future immunizations

Ans: A, B, C Feedback: Different bariatric surgical procedures entail different lifestyle modifications; patients must be well informed about the specific lifestyle changes, eating habits, and bowel habits that may result from a particular procedure. Bariatric surgeries do not influence the future use of immunizations or fertility, though pregnancy should be avoided for 18 months after bariatric surgery.

A nurse is caring for a patient in the late stages of esophageal cancer. The nurse should plan to prevent or address what characteristics of this stage of the disease? Select all that apply. A) Perforation into the mediastinum B) Development of an esophageal lesion C) Erosion into the great vessels D) Painful swallowing E) Obstruction of the esophagus

Ans: A, C, E Feedback: In the later stages of esophageal cancer, obstruction of the esophagus is noted, with possible perforation into the mediastinum and erosion into the great vessels. Painful swallowing and the emergence of a lesion are early signs of esophageal cancer.

The nurse's comprehensive assessment of a patient includes inspection for signs of oral cancer. What assessment finding is most characteristic of oral cancer in its early stages? A) Dull pain radiating to the ears and teeth B) Presence of a painless sore with raised edges C) Areas of tenderness that make chewing difficult D) Diffuse inflammation of the buccal mucosa

Ans: B Feedback: Malignant lesions of the oral cavity are most often painless lumps or sores with raised borders. Because they do not bother the patient, delay in seeking treatment occurs frequently, and negatively affects prognosis. Dull pain radiating to the ears and teeth is characteristic of malocclusion. Inflammation of the buccal mucosa causes discomfort and often occurs as a side effect of chemotherapy. Tenderness resulting in pain on chewing may be associated with gingivitis, abscess, irritation from dentures, and other causes. Pain related to oral cancer is a late symptom.

A staff educator is reviewing the causes of gastroesophageal reflux disease (GERD) with new staff nurses. What area of the GI tract should the educator identify as the cause of reduced pressure associated with GERD? A) Pyloric sphincter B) Lower esophageal sphincter C) Hypopharyngeal sphincter D) Upper esophageal sphincter

Ans: B Feedback: The lower esophageal sphincter, also called the gastroesophageal sphincter or cardiac sphincter, is located at the junction of the esophagus and the stomach. An incompetent lower esophageal sphincter allows reflux (backward flow) of gastric contents. The upper esophageal sphincter and the hypopharyngeal sphincter are synonymous and are not responsible for the manifestations of GERD. The pyloric sphincter exists between the stomach and the duodenum.

A nurse is caring for a patient who has just had a rigid fixation of a mandibular fracture. When planning the discharge teaching for this patient, what would the nurse be sure to include? A) Increasing calcium intake to promote bone healing B) Avoiding chewing food for the specified number of weeks after surgery C) Techniques for managing parenteral nutrition in the home setting D) Techniques for managing a gastrostomy

Ans: B Feedback: The patient who has had rigid fixation should be instructed not to chew food in the first 1 to 4 weeks after surgery. A liquid diet is recommended, and dietary counseling should be obtained to ensure optimal caloric and protein intake. Increased calcium intake will not have an appreciable effect on healing. Enteral and parenteral nutrition are rarely necessary.

Results of a patient barium swallow suggest that the patient has GERD. The nurse is planning health education to address the patient's knowledge of this new diagnosis. Which of the following should the nurse encourage? A) Eating several small meals daily rather than 3 larger meals B) Keeping the head of the bed slightly elevated C) Drinking carbonated mineral water rather than soft drinks D) Avoiding food or fluid intake after 6:00 p.m.

Ans: B Feedback: The patient with GERD is encouraged to elevate the head of the bed on 6- to 8-inch (15- to 20-cm) blocks. Frequent meals are not specifically encouraged and the patient should avoid food and fluid within 2 hours of bedtime. All carbonated beverages should be avoided.

A patient who had a hemiglossectomy earlier in the day is assessed postoperatively, revealing a patent airway, stable vital signs, and no bleeding or drainage from the operative site. The nurse notes the patient is alert. What is the patient's priority need at this time? A) Emotional support from visitors and staff B) An effective means of communicating with the nurse C) Referral to a speech therapist D) Dietary teaching focused on consistency of food and frequency of feedings

Ans: B Feedback: Verbal communication may be impaired by radical surgery for oral cancer. It is therefore vital to assess the patients ability to communicate in writing before surgery. Emotional support and dietary teaching are critical aspects of the plan of care; however, the patients ability to communicate would be essential for both. Referral to a speech therapist will be required as part of the patients rehabilitation; however, it is not a priority at this particular time. Communication with the nurse is crucial for the delivery of safe and effective care.

A client has had frequent watery stools (diarrhea) for an extended period of time. The client also has decreased skin turgor and dark urine. Based on these data, which nursing diagnosis would be appropriate? A) Imbalanced Nutrition: Less than Body Requirements B) Deficient Fluid Volume C) Impaired Tissue Integrity D) Impaired Urinary Elimination

Ans: B Feedback: Bowel elimination problems may also affect other areas of human functioning. For example, excessive diarrhea causes loss of body fluid, with resulting decreased skin turgor and concentrated urine. Deficient Fluid Volume is an appropriate nursing diagnosis based on the data.

A nurse is providing discharge instructions for a client with a new colostomy. Which of the following is a recommended guideline for long-term ostomy care? A) During the first six to eight weeks after surgery, eat foods high in fiber. B) Drink at least two quarts of fluids, preferably water, daily. C) Use enteric-coated or sustained-release medications if needed. D) Use a mild laxative if needed.

Ans: B Feedback: During the first six to eight weeks after surgery, the nurse should encourage the client with an ostomy to avoid foods high in fiber (e.g., foods with skins, seeds, and shells) as well as any other foods that cause diarrhea or excessive flatus. By gradually adding new foods, the ostomy client can progress to a normal diet. The nurse should urge clients to drink at least two quarts of fluids, preferably water, daily. The use of liquid, chewable, or injectable forms rather than long-acting, enteric-coated, or sustained-release medications is recommended. Laxatives and enemas are dangerous because they may cause severe fluid and electrolyte imbalance.

A client is having difficulty having a bowel movement on the bedpan. What is the physiologic reason for this problem? A) It is painful to sit on a bedpan. B) The position does not facilitate downward pressure. C) The position encourages the Valsalva maneuver. D) The cause is unknown and requires further study.

Ans: B Feedback: Most people assume the squatting or slightly forward-sitting position with the thighs flexed to defecate. These positions result in increased pressure on the abdomen and downward pressure on the rectum to facilitate defecation. Obtaining the same results when seated on a bedpan is difficult.

During a home visit, the nurse learns that the client ensures a daily bowel movement with the help of laxatives. The client feels that deviation from a bowel movement every day is unhealthy. Which nursing diagnosis would the nurse most likely identify? A) Constipation B) Perceived constipation C) Risk of constipation D) Bowel incontinence

Ans: B Feedback: The most appropriate nursing diagnosis for the client is perceived constipation, because the client has made a self-diagnosis of constipation and ensures a daily bowel movement through the abuse of laxatives. Constipation may be diagnosed in a client if there is a decrease in the normal frequency of defecation accompanied by a difficult or incomplete passage of stool (and/or passage of excessively hard, dry stool). Risk of constipation can be diagnosed if a client exhibits factors that predispose him or her for developing constipation. Bowel incontinence would be indicated if the client was experiencing an involuntary passage of stool.

A nurse in an oral surgery practice is working with a patient scheduled for removal of an abscessed tooth. When providing discharge education, the nurse should recommend which of the following actions? A) Rinse the mouth with alcohol before bedtime for the next 7 days. B) Use warm saline to rinse the mouth as needed. C) Brush around the area with a firm toothbrush to prevent infection. D) Use a toothpick to dislodge any debris that gets lodged in the socket.

Ans: B Feedback: The patient should be assessed for bleeding after the tooth is extracted. The mouth can be rinsed with warm saline to keep the area clean. A firm toothbrush or toothpick could injure the tissues around the extracted area. Alcohol would injure tissues that are healing.

A nurse is caring for a patient who has just been diagnosed with a peptic ulcer. When teaching the patient about his new diagnosis, how should the nurse best describe a peptic ulcer? A) Inflammation of the lining of the stomach B) Erosion of the lining of the stomach or intestine C) Bleeding from the mucosa in the stomach D) Viral invasion of the stomach wall

Ans: B Feedback: A peptic ulcer is erosion of the lining of the stomach or intestine. Peptic ulcers are often accompanied by bleeding and inflammation, but these are not the definitive characteristics.

A patient with a history of peptic ulcer disease has presented to the emergency department (ED) in distress. What assessment finding would lead the ED nurse to suspect that the patient has a perforated ulcer? A) The patient has abdominal bloating that developed rapidly. B) The patient has a rigid, boardlike abdomen that is tender. C) The patient is experiencing intense lower right quadrant pain. D) The patient is experiencing dizziness and confusion with no apparent hemodynamic changes.

Ans: B Feedback: An extremely tender and rigid (boardlike) abdomen is suggestive of a perforated ulcer. None of the other listed signs and symptoms is suggestive of a perforated ulcer.

A patient has recently received a diagnosis of gastric cancer; the nurse is aware of the importance of assessing the patient's level of anxiety. Which of the following actions is most likely to accomplish this? A) The nurse gauges the patients response to hypothetical outcomes. B) The patient is encouraged to express fears openly. C) The nurse provides detailed and accurate information about the disease. D) The nurse closely observes the patients body language.

Ans: B Feedback: Encouraging the patient to discuss his or her fears and anxieties is usually the best way to assess a patient's anxiety. Presenting hypothetical situations is a surreptitious and possibly inaccurate way of assessing anxiety. Observing body language is part of assessment, but it is not the complete assessment. Presenting information may alleviate anxiety for some patients, but it is not an assessment.

A patient who is obese is exploring bariatric surgery options and presented to a bariatric clinic for preliminary investigation. The nurse interviews the patient, analyzing and documenting the data. Which of the following nursing diagnoses may be a contraindication for bariatric surgery? A) Disturbed Body Image Related to Obesity B) Deficient Knowledge Related to Risks and Expectations of Surgery C) Anxiety Related to Surgery D) Chronic Low Self-Esteem Related to Obesity

Ans: B Feedback: It is expected that patients seeking bariatric surgery may have challenges with body image and self-esteem related to their obesity. Anxiety is also expected when facing surgery. However, if the patient's knowledge remains deficient regarding the risks and realistic expectations for surgery, this may show that the patient is not an appropriate surgical candidate.

A nurse is assessing a patient who has peptic ulcer disease. The patient requests more information about the typical causes of Helicobacter pylori infection. What would it be appropriate for the nurse to instruct the patient? A) Most affected patients acquired the infection during international travel. B) Infection typically occurs due to ingestion of contaminated food and water. C) Many people possess genetic factors causing a predisposition to H. pylori infection. D) The H. pylori microorganism is endemic in warm, moist climates.

Ans: B Feedback: Most peptic ulcers result from infection with the gram-negative bacteria H. pylori, which may be acquired through ingestion of food and water. The organism is endemic to all areas of the United States. Genetic factors have not been identified.

A patient comes to the bariatric clinic to obtain information about bariatric surgery. The nurse assesses the obese patient knowing that in addition to meeting the criterion of morbid obesity, a candidate for bariatric surgery must also demonstrate what? A) Knowledge of the causes of obesity and its associated risks B) Adequate understanding of required lifestyle changes C) Positive body image and high self-esteem D) Insight into why past weight loss efforts failed

Ans: B Feedback: Patients seeking bariatric surgery should be free of serious mental disorders and motivated to comply with lifestyle changes related to eating patterns, dietary choices, and elimination. While assessment of knowledge about causes of obesity and its associated risks as well as insight into the reasons why previous diets have been ineffective are included in the client's plan of care, these do not predict positive client outcomes following bariatric surgery. Most obese patients have an impaired body image and alteration in self-esteem. An obese patient with a positive body image would be unlikely to seek this surgery unless he or she was experiencing significant comorbidities.

A patient has been prescribed orlistat (Xenical) for the treatment of obesity. When providing relevant health education for this patient, the nurse should ensure the patient is aware of what potential adverse effect of treatment? A) Bowel incontinence B) Flatus with oily discharge C) Abdominal pain D) Heat intolerance

Ans: B Feedback: Side effects of orlistat include increased frequency of bowel movements, gas with oily discharge, decreased food absorption, decreased bile flow, and decreased absorption of some vitamins. This drug does not cause bowel incontinence, abdominal pain, or heat intolerance.

A nurse is providing care for a patient who is postoperative day 2 following gastric surgery. The nurse's assessment should be planned in light of the possibility of what potential complications? Select all that apply. A) Malignant hyperthermia B) Atelectasis C) Pneumonia D) Metabolic imbalances E) Chronic gastritis

Ans: B, C, D Feedback: After surgery, the nurse assesses the patient for complications secondary to the surgical intervention, such as pneumonia, atelectasis, or metabolic imbalances resulting from the GI disruption. Malignant hyperthermia is an intraoperative complication. Chronic gastritis is not a surgical complication.

A patient who underwent surgery for esophageal cancer is admitted to the critical care unit following postanesthetic recovery. Which of the following should be included in the patient's immediate postoperative plan of care? A) Teaching the patient to self-suction B) Performing chest physiotherapy to promote oxygenation C) Positioning the patient to prevent gastric reflux D) Providing a regular diet as tolerated

Ans: C Feedback: After recovering from the effects of anesthesia, the patient is placed in a low Fowlers position, and later in a Fowlers position, to help prevent reflux of gastric secretions. The patient is observed carefully for regurgitation and dyspnea because a common postoperative complication is aspiration pneumonia. In this period of recovery, self-suctioning is also not likely realistic or safe. Chest physiotherapy is contraindicated because of the risk of aspiration. Nutrition is prioritized, but a regular diet is contraindicated in the immediate recovery from esophageal surgery.

A patient has undergone surgery for oral cancer and has just been extubated in postanesthetic recovery. What nursing action best promotes comfort and facilitates spontaneous breathing for this patient? A) Placing the patient in a left lateral position B) Administering opioids as ordered C) Placing the patient in Fowler's position D) Teaching the patient to use the patient-controlled analgesia (PCA) system

Ans: C Feedback: After the endotracheal tube or airway has been removed and the effects of the anesthesia have worn off, the patient may be placed in Fowlers position to facilitate breathing and promote comfort. Lateral positioning does not facilitate oxygenation or comfort. Medications do not facilitate spontaneous breathing.

A community health nurse serves a diverse population. What individual would likely face the highest risk for parotitis? A) A patient who is receiving intravenous antibiotic therapy in the home setting B) A patient who has a chronic venous ulcer C) An older adult whose medication regimen includes an anticholinergic D) A patient with poorly controlled diabetes who receives weekly wound care

Ans: C Feedback: Elderly, acutely ill, or debilitated people with decreased salivary flow from general dehydration or medications are at high risk for parotitis. Anticholinergic medications inhibit saliva production. Antibiotics, diabetes, and wounds are not risk factors for parotitis.

A nurse is assessing a patient who has just been admitted to the postsurgical unit following surgical resection for the treatment of oropharyngeal cancer. What assessment should the nurse prioritize? A) Assess ability to clear oral secretions. B) Assess for signs of infection. C) Assess for a patent airway. D) Assess for ability to communicate.

Ans: C Feedback: Postoperatively, the nurse assesses for a patent airway. The patients ability to manage secretions has a direct bearing on airway patency. However, airway patency is the overarching goal. This immediate physiologic need is prioritized over communication, though this is an important consideration. Infection is not normally a threat in the immediate postoperative period.

A patient with cancer of the tongue has had a radical neck dissection. What nursing assessment would be a priority for this patient? A) Presence of acute pain and anxiety B) Tissue integrity and color of the operative site C) Respiratory status and airway clearance D) Self-esteem and body image

Ans: C Feedback: Postoperatively, the patient is assessed for complications such as altered respiratory status, wound infection, and hemorrhage. The other assessments are part of the plan of care for a patient who has had a radical neck dissection, but are not the nurses chief priority.

A patient who has had a radical neck dissection is being prepared for discharge. The discharge plan includes referral to an outpatient rehabilitation center for physical therapy. What would the goals of physical therapy for this patient include? A) Muscle training to relieve dysphagia B) Relieving nerve paralysis in the cervical plexus C) Promoting maximum shoulder function D) Alleviating achalasia by decreasing esophageal peristalsis

Ans: C Feedback: Shoulder drop occurs as a result of radical neck dissection. Shoulder function can be improved by rehabilitation exercises. Rehabilitation would not be initiated until the patients neck incision and graft, if present, were sufficiently healed. Nerve paralysis in the cervical plexus and other variables affecting swallowing would be managed by a speech therapist rather than a physical therapist.

A patient has undergone rigid fixation for the correction of a mandibular fracture suffered in a fight. What area of care should the nurse prioritize when planning this patient's discharge education? A) Resumption of activities of daily living B) Pain control C) Promotion of adequate nutrition D) Strategies for promoting communication

Ans: C Feedback: The patient who has had rigid fixation should be instructed not to chew food in the first 1 to 4 weeks after surgery. A liquid diet is recommended, and dietary counseling should be obtained to ensure optimal caloric and protein intake. The nature of this surgery threatens the patients nutritional status; this physiologic need would likely supersede the resumption of ADLs. Pain should be under control prior to discharge and communication is not precluded by this surgery.

A client tells the nurse, "I increased my fiber, but I am very constipated." What further information does the nurse need to tell the client? A) "Just give it a few more days and you should be fine." B) "Well, that shouldn't happen. Let me recommend a good laxative for you." C) "When you increase fiber in your diet, you also need to increase liquids." D) "I will tell the doctor you are having problems; maybe he can help."

Ans: C Feedback: A combination of high-fiber foods, 8 to 10 glasses of water a day, and exercise has been shown to be as effective as medications in controlling constipation. Caution the client to avoid increasing fiber intake without drinking enough fluids because this can lead to a bowel obstruction.

Then nurse is preparing to apply a fecal incontinence pouch. Arrange the following steps in the correct order. 1. Cleanse entire perianal area and pat dry. 2. Apply skin protectant and allow it to dry. 3. Separate buttocks and apply the pouch to the anal area. 4. Attach the pouch to a urinary drainage bag. 5. Hang the drainage bag below the patient. A) 2, 3, 4, 5, 1 B) 3, 4, 5, 1, 2 C) 1, 2, 3, 4, 5 D) 5, 4, 3, 2, 1

Ans: C Feedback: A nurse would not be able to determine if the entire intestinal tract is clear.

Which type of stool would the nurse assess in a client with an illness that causes the stool to pass through the large intestine quickly? A) Hard, formed B) Black, tarry C) Soft, watery D) Dry, odorous

Ans: C Feedback: About 800 to 1,000 mL of liquid is absorbed daily by the large intestine. When absorption does not occur properly, such as when the waste products pass through the large intestine rapidly, the stool is soft and watery.

A physician orders a retention enema for a client to destroy intestinal parasites. Which of the following enemas would be indicated for this client? A) Oil retention enema B) Carminative enema C) Anthelmintic enema D) Nutritive enema

Ans: C Feedback: Anthelmintic enemas are administered to destroy intestinal parasites. Oil retention enemas help to lubricate the stool and intestinal mucosa, making defecation easier. Carminative enemas help to expel flatus from the rectum and relieve distention. Nutritive enemas are administered to replenish fluids and nutrition rectally.

A nurse is ordered to perform digital removal of stool on a client with stool impaction. Which of the following is an appropriate step in this procedure? A) Position the client in supine position as dictated by client comfort and condition. B) Insert generously lubricated finger gently into the anal canal, pointing away from the umbilicus. C) Gently work the finger around and into the hardened mass to break it up and then remove pieces of it. D) Instruct the client not to bear down while extracting feces to prevent vagal response.

Ans: C Feedback: For digital removal of stool: Position the client on the left side (Sims' position), as dictated by client comfort and condition. Generously lubricate index finger with water-soluble lubricant and insert finger gently into anal canal, pointing toward the umbilicus. Gently work the finger around and into the hardened mass to break it up and then remove pieces of it. Instruct the client to bear down, if possible, while extracting feces, which will ease in removal.

A nurse is providing care to a client who has undergone a colonoscopy. Which of the following would be most appropriate for the nurse to do after the procedure? A) Avoid giving solid food B) Administer a laxative to the client C) Monitor for rectal bleeding D) Limit oral fluid intake

Ans: C Feedback: The nurse should monitor the client for rectal bleeding after a colonoscopy. The nurse should provide rest and offer food and fluids as allowed. The evening before the procedure, solid foods are avoided and liquids are encouraged. Laxatives are also given before the procedure.

A nurse is caring for a client who is postoperative Day 1 for a temporary colostomy. The nurse assesses no feces in the collection bag. What should the nurse do next? A) Notify the physician immediately. B) Ask another nurse to check her findings. C) Nothing; this is normal. D) Recheck the bag in two hours.

Ans: C Feedback: Typically, a colostomy does not produce drainage or feces until normal peristalsis returns after surgery, usually within two to five days.

An infant has had diarrhea for several days. What assessments will the nurse make to identify risks from the diarrhea? A) Heart tones B) Lung sounds C) Skin turgor D) Activity level

Ans: C Feedback: When infants and children become ill, they lose most fluids from their extracellular compartment, which quickly leads to dehydration. The nurse would assess skin turgor to identify this problem.

A nurse is addressing the prevention of esophageal cancer in response to a question posed by a participant in a health promotion workshop. What action has the greatest potential to prevent esophageal cancer? A) Promotion of a nutrient-dense, low-fat diet B) Annual screening endoscopy for patients over 50 with a family history of esophageal cancer C) Early diagnosis and treatment of gastroesophageal reflux disease D) Adequate fluid intake and avoidance of spicy foods

Ans: C Feedback: There are numerous risk factors for esophageal cancer but chronic esophageal irritation or GERD is among the most significant. This is a more significant risk factor than dietary habits. Screening endoscopies are not recommended solely on the basis of family history.

A nurse is preparing to discharge a patient after recovery from gastric surgery. What is an appropriate discharge outcome for this patient? A) The patient's bowel movements maintain a loose consistency. B) The patient is able to tolerate three large meals a day. C) The patient maintains or gains weight. D) The patient consumes a diet high in calcium.

Ans: C Feedback: Expected outcomes for the patient following gastric surgery include ensuring that the patient is maintaining or gaining weight (patient should be weighed daily), experiencing no excessive diarrhea, and tolerating six small meals a day. Patients may require vitamin B12 supplementation by the intramuscular route and do not require a diet excessively rich in calcium.

A nurse is providing patient education for a patient with peptic ulcer disease secondary to chronic nonsteroidal anti-inflammatory drug (NSAID) use. The patient has recently been prescribed misoprostol (Cytotec). What would the nurse be most accurate in informing the patient about the drug? A) It reduces the stomachs volume of hydrochloric acid B) It increases the speed of gastric emptying C) It protects the stomachs lining D) It increases lower esophageal sphincter pressure

Ans: C Feedback: Misoprostol is a synthetic prostaglandin that, like prostaglandin, protects the gastric mucosa. NSAIDs decrease prostaglandin production and predispose the patient to peptic ulceration. Misoprostol does not reduce gastric acidity, improve emptying of the stomach, or increase lower esophageal sphincter pressure.

A nurse is completing a health history on a patient whose diagnosis is chronic gastritis. Which of the data should the nurse consider most significantly related to the etiology of the patient's health problem? A) Consumes one or more protein drinks daily. B) Takes over-the-counter antacids frequently throughout the day. C) Smokes one pack of cigarettes daily. D) Reports a history of social drinking on a weekly basis.

Ans: C Feedback: Nicotine reduces secretion of pancreatic bicarbonate, which inhibits neutralization of gastric acid and can underlie gastritis. Protein drinks do not result in gastric inflammation. Antacid use is a response to experiencing symptoms of gastritis, not the etiology of gastritis. Alcohol ingestion can lead to gastritis; however, this generally occurs in patients with a history of consumption of alcohol on a daily basis.

A nurse is providing anticipatory guidance to a patient who is preparing for bariatric surgery. The nurse learns that the patient is anxious about numerous aspects of the surgery. What intervention is most appropriate to alleviate the patient's anxiety? A) Emphasize the fact that bariatric surgery has a low risk of complications. B) Encourage the patient to focus on the benefits of the surgery. C) Facilitate the patients contact with a support group. D) Obtain an order for a PRN benzodiazepine.

Ans: C Feedback: Support groups can be highly beneficial in relieving preoperative and postoperative anxiety and in promoting healthy coping. This is preferable to antianxiety medications. Downplaying the risks of surgery or focusing solely on the benefits is a simplistic and patronizing approach.

A patient who underwent gastric banding 3 days ago is having her diet progressed on a daily basis. Following her latest meal, the patient complains of dizziness and palpitations. Inspection reveals that the patient is diaphoretic. What is the nurse's best action? A) Insert a nasogastric tube promptly. B) Reposition the patient supine. C) Monitor the patient closely for further signs of dumping syndrome. D) Assess the patient for signs and symptoms of aspiration.

Ans: C Feedback: The patient's symptoms are characteristic of dumping syndrome, which results in a sensation of fullness, weakness, faintness, dizziness, palpitations, diaphoresis, cramping pains, and diarrhea. Aspiration is a less likely cause for the patient's symptoms. Supine positioning will likely exacerbate the symptoms and insertion of an NG tube is contraindicated due to the nature of the patient's surgery.

A patient who is obese has been unable to lose weight successfully using lifestyle modifications and has mentioned the possibility of using weight-loss medications. What should the nurse teach the patient about pharmacologic interventions for the treatment of obesity? A) "Weight loss drugs have many side effects, and most doctors think they'll all be off the market in a few years." B) "There used to be a lot of hope that medications would help people lose weight, but it's been shown to be mostly a placebo effect." C) "Medications can be helpful, but few people achieve and maintain their desired weight loss with medications alone." D) "Medications are rapidly become the preferred method of weight loss in people for whom diet and exercise have not worked."

Ans: C Feedback: Though antiobesity drugs help some patients lose weight, their use rarely results in loss of more than 10% of total body weight. Patients are consequently unlikely to attain their desired weight through medication alone. They are not predicted to disappear from the market and results are not attributed to a placebo effect.

A patient is undergoing diagnostic testing for a tumor of the small intestine. What are the most likely symptoms that prompted the patient to first seek care? A) Hematemesis and persistent sensation of fullness B) Abdominal bloating and recurrent constipation C) Intermittent pain and bloody stool D) Unexplained bowel incontinence and fatty stools

Ans: C Feedback: When the patient is symptomatic from a tumor of the small intestine, benign tumors often present with intermittent pain. The next most common presentation is occult bleeding. The other listed signs and symptoms are not normally associated with the presentation of small intestinal tumors.

A nurse is providing oral care to a patient who is comatose. What action best addresses the patient's risk of tooth decay and plaque accumulation? A) Irrigating the mouth using a syringe filled with a bacteriocidal mouthwash B) Applying a water-soluble gel to the teeth and gums C) Wiping the teeth and gums clean with a gauze pad D) Brushing the patient's teeth with a toothbrush and small amount of toothpaste

Ans: D Feedback: Application of mechanical friction is the most effective way to cleanse the patients mouth. If the patient is unable to brush teeth, the nurse may brush them, taking precautions to prevent aspiration; or as a substitute, the nurse can achieve mechanical friction by wiping the teeth with a gauze pad. Bacteriocidal mouthwash does reduce plaque-causing bacteria; however, it is not as effective as application of mechanical friction. Water-soluble gel may be applied to lubricate dry lips, but it is not part of oral care.

A nurse is performing health education with a patient who has a history of frequent, serious dental caries. When planning educational interventions, the nurse should identify a risk for what nursing diagnosis? A) Ineffective Tissue Perfusion B) Impaired Skin Integrity C) Aspiration D) Imbalanced Nutrition: Less Than Body Requirements

Ans: D Feedback: Because digestion normally begins in the mouth, adequate nutrition is related to good dental health and the general condition of the mouth. Any discomfort or adverse condition in the oral cavity can affect a persons nutritional status. Dental caries do not typically affect the patients tissue perfusion or skin integrity. Aspiration is not a likely consequence of dental caries.

A patient has been diagnosed with a malignancy of the oral cavity and is undergoing oncologic treatment. The oncologic nurse is aware that the prognosis for recovery from head and neck cancers is often poor because of what characteristic of these malignancies? A) Radiation therapy often results in secondary brain tumors. B) Surgical complications are exceedingly common. C) Diagnosis rarely occurs until the cancer is endstage. D) Metastases are common and respond poorly to treatment.

Ans: D Feedback: Deaths from malignancies of the head and neck are primarily attributable to local-regional metastasis to the cervical lymph nodes in the neck. This often occurs by way of the lymphatics before the primary lesion has been treated. This local-regional metastasis is not amenable to surgical resection and responds poorly to chemotherapy and radiation therapy. This high mortality rate is not related to surgical complications, late diagnosis, or the development of brain tumors.

An emergency department nurse is admitting a 3-year-old brought in after swallowing a piece from a wooden puzzle. The nurse should anticipate the administration of what medication in order to relax the esophagus to facilitate removal of the foreign body? A) Haloperidol B) Prostigmine C) Epinephrine D) Glucagon

Ans: D Feedback: Glucagon is administered prior to removal of a foreign body because it relaxes the smooth muscle of the esophagus, facilitating insertion of the endoscope. Haloperidol is an antipsychotic drug and is not indicated. Prostigmine is prescribed for patients with myastheniagravis. It increases muscular contraction, an effect opposite that which is desired to facilitate removal of the foreign body. Epinephrine is indicated in asthma attack and bronchospasm.

A nurse is providing care for a patient whose neck dissection surgery involved the use of a graft. When assessing the graft, the nurse should prioritize data related to what nursing diagnosis? A) Risk for Disuse Syndrome B) Unilateral Neglect C) Risk for Trauma D) Ineffective Tissue Perfusion

Ans: D Feedback: Grafted skin is highly vulnerable to inadequate perfusion and subsequent ischemia and necrosis. This is a priority over chronic pain, which is unlikely to be a long-term challenge. Neglect and disuse are not risks related to the graft site.

A radial graft is planned in the treatment of a patient's oropharyngeal cancer. In order to ensure that the surgery will be successful, the care team must perform what assessment prior to surgery? A) Assessing function of cranial nerves V, VI, and IX B) Assessing for a history of GERD C) Assessing for signs or symptoms of atherosclerosis D) Assessing the patency of the ulnar artery

Ans: D Feedback: If a radial graft is to be performed, an Allen test on the donor arm must be performed to ensure that the ulnar artery is patent and can provide blood flow to the hand after removal of the radial artery. The success of this surgery is not primarily dependent on CN function or the absence of GERD and atherosclerosis.

A nurse is caring for a patient who is postoperative day 1 following neck dissection surgery. The nurse is performing an assessment of the patient and notes the presence of high-pitched adventitious sounds over the patient's trachea on auscultation. The patient's oxygen saturation is 90% by pulse oximetry with a respiratory rate of 31 breaths per minute. What is the nurse's most appropriate action? A) Encourage the patient to perform deep breathing and coughing exercises hourly. B) Reposition the patient into a prone or semi-Fowler's position and apply supplementary oxygen by nasal cannula. C) Activate the emergency response system. D) Report this finding promptly to the physician and remain with the patient.

Ans: D Feedback: In the immediate postoperative period, the nurse assesses for stridor (coarse, high-pitched sound on inspiration) by listening frequently over the trachea with a stethoscope. This finding must be reported immediately because it indicates obstruction of the airway. The patients current status does not warrant activation of the emergency response system, and encouraging deep breathing and repositioning the patient are inadequate responses.

A nurse is caring for a patient who is acutely ill and has included vigilant oral care in the patient's plan of care. Why are patients who are ill at increased risk for developing dental caries? A) Hormonal changes brought on by the stress response cause an acidic oral environment B) Systemic infections frequently migrate to the teeth C) Hydration that is received intravenously lacks fluoride D) Inadequate nutrition and decreased saliva production can cause cavities

Ans: D Feedback: Many ill patients do not eat adequate amounts of food and therefore produce less saliva, which in turn reduces the natural cleaning of the teeth. Stress response is not a factor, infections generally do not attack the enamel of the teeth, and the fluoride level of the patient is not significant in the development of dental caries in the ill patient.

The following foods are a part of a client's daily diet: high-fiber cereals, fruits, vegetables, 2,500 mL of fluids. What would the nurse tell the client to change? A) Decrease high-fiber foods B) Decrease amount of fluids C) Omit fruits if eating vegetables D) Nothing; this is a good diet

Ans: D Feedback: A high-fiber diet and a daily fluid intake of 2,500 to 3,000 mL of fluids facilitate bowel elimination. Intake of the foods described makes the feces more bulky, so they move through the intestine more quickly. The stool is softer and the time to absorb toxins is decreased (toxins are believed to have a role in the development of colon cancer).

A nurse assessing a client with an ostomy appliance documents the condition "prolapse" in the client chart and notifies the physician. Which of the following statements describes this condition? A) The peristomal skin is excoriated or irritated because the appliance is cut too large. B) The system has leaks or poor adhesion leading to noticeable odor. C) The bag continues to come loose and become inverted. D) The stoma is protruding into the bag and may become twisted.

Ans: D Feedback: During prolapse, the stoma is protruding into the bag. The nurse should have the client rest for 30 minutes and, if stoma is not back to normal size within that time, notify the physician. If stoma stays prolapsed, it may twist, resulting in impaired circulation to the stoma.

A client who has been on a medication that caused diarrhea is now off the medication. What could the nurse suggest to promote the return of normal flora? A) Stool-softening laxatives, such as Colace B) Increasing fluid intake to 3,000 mL/day C) Drinking fluids with a high sugar content D) Eating fermented products, such as yogurt

Ans: D Feedback: Some medications, such as antibiotics, may destroy normal intestinal flora and cause diarrhea. To promote the return of normal flora, the nurse can recommend an intake of fermented dairy products, such as buttermilk or yogurt.

A nurse is assessing the stoma of a client with an ostomy. What would the nurse assess in a normal, healthy stoma? A) Pallor B) Purple-blue C) Irritation and bleeding D) Dark red and moist

Ans: D Feedback: The ostomy stoma should be dark pink to red and moist. Abnormal findings include paleness (possible anemia), purple-blue color (possible ischemia), or bleeding.

A nurse is conducting an abdominal assessment. What is the rationale for palpating the abdomen last in the sequence when conducting an abdominal assessment? A) It is the most painful assessment method B) It is the most embarrassing assessment method C) To allow time for the examiner's hands to warm D) It disturbs normal peristalsis and bowel motility

Ans: D Feedback: The sequence for abdominal assessment is inspection, auscultation, percussion, and palpation. Inspection and auscultation are performed before palpation because palpation may disturb normal peristalsis and bowel motility.

A patient seeking care because of recurrent heartburn and regurgitation is subsequently diagnosed with a hiatal hernia. Which of the following should the nurse include in health education? A) "Drinking beverages after your meal, rather than with your meal, may bring some relief." B) "It's best to avoid dry foods, such as rice and chicken, because they're harder to swallow." C) "Many patients obtain relief by taking over-the-counter antacids 30 minutes before eating." D) "Instead of eating three meals a day, try eating smaller amounts more often."

Ans: D Feedback: Management for a hiatal hernia includes frequent, small feedings that can pass easily through the esophagus. Avoiding beverages and particular foods or taking OTC antacids are not noted to be beneficial.

A patient with a peptic ulcer disease has had metronidazole (Flagyl) added to his current medication regimen. What health education related to this medication should the nurse provide? A) Take the medication on an empty stomach. B) Take up to one extra dose per day if stomach pain persists. C) Take at bedtime to mitigate the effects of drowsiness. D) Avoid drinking alcohol while taking the drug.

Ans: D Feedback: Alcohol must be avoided when taking Flagyl and the medication should be taken with food. This drug does not cause drowsiness and the dose should not be adjusted by the patient.

A patient has been diagnosed with peptic ulcer disease and the nurse is reviewing his prescribed medication regimen with him. What is currently the most commonly used drug regimen for peptic ulcers? A) Bismuth salts, antivirals, and histamine-2 (H2) antagonists B) H2 antagonists, antibiotics, and bicarbonate salts C) Bicarbonate salts, antibiotics, and ZES D) Antibiotics, proton pump inhibitors, and bismuth salts

Ans: D Feedback: Currently, the most commonly used therapy for peptic ulcers is a combination of antibiotics, proton pump inhibitors, and bismuth salts that suppress or eradicate H. pylori. H2 receptor antagonists are used to treat NSAID-induced ulcers and other ulcers not associated with H. pylori infection, but they are not the drug of choice. Bicarbonate salts are not used. ZES is the Zollinger-Ellison syndrome and not a drug.

A patient has just been diagnosed with acute gastritis after presenting in distress to the emergency department with abdominal symptoms. What would be the nursing care most needed by the patient at this time? A) Teaching the patient about necessary nutritional modification B) Helping the patient weigh treatment options C) Teaching the patient about the etiology of gastritis D) Providing the patient with physical and emotional support

Ans: D Feedback: For acute gastritis, the nurse provides physical and emotional support and helps the patient manage the symptoms, which may include nausea, vomiting, heartburn, and fatigue. The scenario describes a newly diagnosed patient; teaching about the etiology of the disease, lifestyle modifications, or various treatment options would be best provided at a later time.

18. A patient has experienced symptoms of dumping syndrome following bariatric surgery. To what physiologic phenomenon does the nurse attribute this syndrome? A) Irritation of the phrenic nerve due to diaphragmatic pressure B) Chronic malabsorption of iron and vitamins A and C C) Reflux of bile into the distal esophagus D) A sudden release of peptides

Ans: D Feedback: For many years, it had been theorized that the hypertonic gastric food boluses that quickly transit into the intestines drew extracellular fluid from the circulating blood volume into the small intestines to dilute the high concentration of electrolytes and sugars, resulting in symptoms. Now, it is thought that this rapid transit of the food bolus from the stomach into the small intestines instead causes a rapid and exuberant release of metabolic peptides that are responsible for the symptoms of dumping syndrome. It is not a result of phrenic nerve irritation, malabsorption, or bile reflux.

A nurse is caring for a patient hospitalized with an exacerbation of chronic gastritis. What health promotion topic should the nurse emphasize? A) Strategies for maintaining an alkaline gastric environment B) Safe technique for self-suctioning C) Techniques for positioning correctly to promote gastric healing D) Strategies for avoiding irritating foods and beverages

Ans: D Feedback: Measures to help relieve pain include instructing the patient to avoid foods and beverages that may be irritating to the gastric mucosa and instructing the patient about the correct use of medications to relieve chronic gastritis. An alkaline gastric environment is neither possible nor desirable. There is no plausible need for self-suctioning. Positioning does not have a significant effect on the presence or absence of gastric healing.

A community health nurse is preparing for an initial home visit to a patient discharged following a total gastrectomy for treatment of gastric cancer. What would the nurse anticipate that the plan of care is most likely to include? A) Enteral feeding via gastrostomy tube (G tube) B) Gastrointestinal decompression by nasogastric tube C) Periodic assessment for esophageal distension D) Monthly administration of injections of vitamin B12

Ans: D Feedback: Since vitamin B12 is absorbed in the stomach, the patient requires vitamin B12 replacement to prevent pernicious anemia. A gastrectomy precludes the use of a G tube. Since the stomach is absent, a nasogastric tube would not be indicated. As well, this is not possible in the home setting. Since there is no stomach to act as a reservoir and fluids and nutrients are passing directly into the jejunum, distension is unlikely.

A nurse is admitting a patient diagnosed with late-stage gastric cancer. The patient's family is distraught and angry that she was not diagnosed earlier in the course of her disease. What factor contributes to the fact that gastric cancer is often detected at a later stage? A) Gastric cancer does not cause signs or symptoms until metastasis has occurred. B) Adherence to screening recommendations for gastric cancer is exceptionally low. C) Early symptoms of gastric cancer are usually attributed to constipation. D) The early symptoms of gastric cancer are usually not alarming or highly unusual.

Ans: D Feedback: Symptoms of early gastric cancer, such as pain relieved by antacids, resemble those of benign ulcers and are seldom definitive. Symptoms are rarely a cause for alarm or for detailed diagnostic testing. Symptoms precede metastasis, however, and do not include constipation.

A nurse is caring for a patient who has a diagnosis of GI bleed. During shift assessment, the nurse finds the patient to be tachycardic and hypotensive, and the patient has an episode of hematemesis while the nurse is in the room. In addition to monitoring the patient's vital signs and level of consciousness, what would be a priority nursing action for this patient? A) Place the patient in a prone position. B) Provide the patient with ice water to slow any GI bleeding. C) Prepare for the insertion of an NG tube. D) Notify the physician.

Ans: D Feedback: The nurse must always be alert for any indicators of hemorrhagic gastritis, which include hematemesis (vomiting of blood), tachycardia, and hypotension. If these occur, the physician is notified and the patient's vital signs are monitored as the patient's condition warrants. Putting the patient in a prone position could lead to aspiration. Giving ice water is contraindicated as it would stimulate more vomiting.

A patient is recovering in the hospital following gastrectomy. The nurse notes that the patient has become increasingly difficult to engage and has had several angry outbursts at various staff members in recent days. The nurse's attempts at therapeutic dialogue have been rebuffed. What is the nurse's most appropriate action? A) Ask the patients primary care provider to liaise between the nurse and the patient. B) Delegate care of the patient to a colleague. C) Limit contact with the patient in order to provide privacy. D) Make appropriate referrals to services that provide psychosocial support.

Ans: D Feedback: The nurse should enlist the services of clergy, psychiatric clinical nurse specialists, psychologists, social workers, and psychiatrists, if needed. This is preferable to delegating care, since the patient has become angry with other care providers as well. It is impractical and inappropriate to expect the primary care provider to act as a liaison. It would be inappropriate and unsafe to simply limit contact with the patient.


संबंधित स्टडी सेट्स

Global Sports and National Culture Exam 1

View Set

Programming Fundalmentals II — Test 4

View Set

ACCT 101 Ch. 12 THE STATEMENT OF CASH FLOWS 2017

View Set

Life insurance basic Chapter quiz

View Set

***HURST Review Elevate Q-Cards***

View Set

Module 9 - Conductors and Cables

View Set

Chapter 11: Health and Well-being

View Set

Legal Aspects of Business Chapter 3

View Set